How to Calculate the Magnetic Field at the Origin for a Finite Solenoid?

In summary, the problem is to find the magnitude of the magnetic field at the origin due to a cylindrical shell with a current I distributed uniformly along its z-axis. After trying different approaches, including using Ampere's Law and Biot-Savart Law, the correct solution is found by dividing the cylinder into loops and using the Biot-Savart Law with the current density formula for cylinders. The final result is \frac{\mu_0 I L}{\sqrt{a^2+L^2}} which is less than the field from a solenoid with the same current and length.
  • #1
physicsjones
3
0

Homework Statement


A cylindrical shell of radius a and length 2L is aligned around the z-axis from z= -L ot z = +L. A current I is distributed uniformly on the cylinder and moves around the cylinder's z-axis. Find the magnitude of the magnetic field at the origin.


Homework Equations


Biot-Savart Law [itex] \vec{B} = \frac{\mu_0}{4 \pi} \int\frac{d\vec{I} \times \vec{r}}{r^2}[/itex]
Ampere's Law [itex] \int \vec{B}\dot{} d\vec{l} = \mu_0 I_{enc}[/itex]


The Attempt at a Solution


I've tried this several different ways. First, I tried with ampere's law, the way you would with an infinite solenoid, but that doesn't work since the field isn't perpendicular/constant at the end pieces.

I then tried to use Biot-Savart, but may have done so incorrectly. I got [itex] B = \frac{\mu_0 I}{4 \pi} 2 \pi \int\limits_{-L}^{L} \frac{a}{\sqrt{a^2+z^2}} dz [/itex], which simplifies to (after some Mathematica) [itex] \mu_0 I a \mathrm{arcsinh}(\frac{L}{x}) [/itex], which isn't able to be evaluated at the origin. So I'm stuck.

I've looked at answers like the one given at http://www.phys.uri.edu/~gerhard/PHY204/tsl215.pdf, but have difficulty following and translating them to my problem.
 
Physics news on Phys.org
  • #2
Have you tried dividing the cylinder into loops width dz with current dI?
 
  • #3
Just tried it now
Using Griffith's formula for on-axis field from a current loop (example 5.6 3rd edition)
[itex]B(z) = \frac{\mu_0 I}{2} \frac{a^2}{(a^2+z^2)^{3/2}}[/itex]

I integrate this with respect to z from -L to L and the result is [itex]\frac{\mu_0 I L}{\sqrt{a^2+L^2}}[/itex]
Seem reasonable?
 
  • #4
That sounds reasonable - though I havn't done the maths myself.
The result should be less than if the entire current were in a single loop...

how does it compare to a solenoid with N turns and current I?
 
  • #5
Usually a solenoid has field [itex] \frac{\mu_0 N I}{L}[/itex].
I think that means the units are off in my solution since I essentially have units of [itex] \mu_0 I[/itex] and those are of [itex]\frac{\mu_0 I}{L}[/itex].
I just double checked the integral, and that was evaluated correctly, which means I set it up incorrectly.

I tried it again by by taking the derivative of the above formula to get an expression for dB, and integrating across z, the same as plugging in L and -L to the formula above. Since the only z is squared, this gives 0.

That actually makes so much sense. Since we're at the center of a finite line, the fields from each "ring" are going in opposite directions.

Thanks for your help!
 
  • #6
That actually makes so much sense. Since we're at the center of a finite line, the fields from each "ring" are going in opposite directions.
If the currents are going anticlockwise (viewed looking down the z axis) then field along the z axis due to the ring of current dI radius a at position z (between z and z+dz) is pointing in the +z direction, whether z is above or below the origin. Sketch the picture to see.
http://hyperphysics.phy-astr.gsu.edu/hbase/magnetic/curloo.html
$$\vec B = B\hat k\\ dB = \frac{\mu_0}{4\pi}\frac{2\pi a^2 dI}{(z^2+a^2)^{3/2}}=\frac{\mu_0 I}{4L}\frac{a^2 dz}{(z^2+a^2)^{3/2}}$$... or something.
The difference between the cylinder of current and the solenoid is how the current is distributed.
 
  • Like
Likes 1 person
  • #7
I don't know, what you are calculating. Just use the Biot-Savart Law, of which the notation you wrote in your first posting also doesn't make sense at all.

You need the current density, which is obviously given (in cylinder coordinates) as
[tex]\vec{j}=\frac{I}{2L} \delta(\rho-R) \vec{e}_{\varphi} \Theta(-L<z<L).[/tex]
The Biot-Savart Law reads
[tex]\vec{B}(\vec{r})=\frac{\mu_0}{4 \pi} \int_{\mathbb{R}^3} \mathrm{d}^3 \vec{r}' \vec{j}(\vec{r}') \times \frac{\vec{r}-\vec{r}'}{|\vec{r}-\vec{r}'|^3}.[/tex]
Just evaluate the integral for [itex]\vec{r}=z \vec{e}_z[/itex]! Be aware that the cylinder-coordinate-unit vectors depend on the position. So it's save to rewrite everything in terms of Cartesian components before doing the integral!
 

Related to How to Calculate the Magnetic Field at the Origin for a Finite Solenoid?

1. What is a finite solenoid?

A finite solenoid is a cylindrical coil of wire with a finite length and a uniform current flowing through it. It produces a magnetic field that is similar to that of an ideal solenoid, but only within its finite length.

2. How does a finite solenoid differ from an ideal solenoid?

An ideal solenoid has an infinite length and produces a uniform magnetic field inside its entire length, while a finite solenoid has a finite length and produces a non-uniform magnetic field outside of its length.

3. What are the applications of a finite solenoid?

Finite solenoids are commonly used in electronic devices such as inductors, electromagnets, and transformers. They can also be used in scientific experiments to study magnetic fields and their effects.

4. How is the magnetic field inside a finite solenoid calculated?

The magnetic field inside a finite solenoid can be calculated using the formula B = u0 * n * I, where B is the magnetic field, u0 is the permeability of free space, n is the number of turns per unit length, and I is the current flowing through the solenoid.

5. What factors can affect the magnetic field produced by a finite solenoid?

The magnetic field produced by a finite solenoid can be affected by factors such as the number of turns, the current flowing through the solenoid, the length of the solenoid, and the material of the core (if present). Changes in these factors can alter the strength and uniformity of the magnetic field produced.

Similar threads

  • Advanced Physics Homework Help
Replies
12
Views
1K
  • Introductory Physics Homework Help
Replies
3
Views
187
  • Introductory Physics Homework Help
Replies
7
Views
151
  • Advanced Physics Homework Help
Replies
1
Views
2K
  • Advanced Physics Homework Help
Replies
1
Views
4K
  • Introductory Physics Homework Help
Replies
10
Views
74
  • Advanced Physics Homework Help
Replies
11
Views
1K
  • Introductory Physics Homework Help
Replies
25
Views
1K
Replies
8
Views
729
  • Introductory Physics Homework Help
Replies
4
Views
1K
Back
Top